Sunteți pe pagina 1din 18

2017 MTAP-DepEd-NCR Seminar-Workshop and Convention

Proving Techniques and Mathematical Induction-Answer Key


Direct Proof

Exercises. Use the method of direct proof to prove the following statements.

1. If a is an odd integer, then a2 + 3a + 5 is odd.

Proof. If a is odd, then a = 2k + 1, k ∈ Z. By substitution,

a2 + 3a + 5 =(2k + 1)2 + 3(2k + 1) + 5


=4k 2 + 10k + 8 + 1
=2(2k 2 + 5k + 4) + 1.

Since k ∈ Z, j = 2k 2 + 5k + 4 ∈ Z. By substitution again, a2 + 3a + 5 = 2j + 1 is odd.

2. Suppose x, y ∈ Z. If x is even, then xy is even.

Proof. If x is even, then x = 2l, l ∈ Z. Thus, xy = 2ly is even since ly ∈ Z.

3. If n ∈ Z, then n2 + 3n + 4 is even. (Try cases.)

Proof. If n is even, then n = 2j, j ∈ Z. By substitution, n2 +3n+4 = (2j)2 +3(2j)+4 = 2(2j 2 +6j+2)
is even since 2j 2 + 6j + 2 is an integer. Let n be odd. From item (1), n2 + 3n + 5 is odd. Let k ∈ Z
such that n2 + 3n + 5 = 2k + 1. Observe that n2 + 3n + 4 = n2 + 3n + 5 − 1 = 2k + 1 − 1 = 2k is
even. In any case, n2 + 3n + 4 is even.

4. If two integers have the same parity, then their sum is even. (Try cases.)

Proof. If two integers are both even, then they are of the form 2l and 2k for k, l ∈ Z. The sum of
the two integers is 2l + 2k = 2(k + l) which is even since k + l is an integer. If two integers are both
odd, then they are of the form 2g + 1 and 2h + 1 for g, h ∈ Z. The sum of the two odd integers is
2g + 1 + 2h + 1 = 2(g + h + 1) which is even since g + h + 1 is an integer.

5. Suppose x and y are positive real numbers. If x < y, then x2 < y 2 .

Proof. If x < y, then x · x < x · y and x · y < y · y. By transitivity, x2 < y 2 .

Contrapositive Proof

Exercises. Use the method of contrapositive proof to prove the following statements.

1. Suppose n ∈ Z. If n2 is even, then n is even.

Proof. We show that if n is odd, then n2 is odd. Let n = 2k +1, k ∈ Z. Observe that n2 = (2k +1)2 =
4k 2 + 4k + 1 = 2(2k 2 + 2k) + 1 is odd since j = 2k 2 + 2k ∈ Z and n2 = 2j + 1. Since the contrapositive
of the statement is proved, the statement is now proven.

2. Suppose a, b ∈ Z. If a2 (b2 − 2b) is odd, then a and b are odd.

Proof. The negation of "a and b are odd" is "a or b is even." Our goal is to show that if a or b are
even, then a2 (b2 − 2b) is even. We first observe that a2 (b2 − 2b) = ab(ab − 2a). If a is even, then
a = 2m, m ∈ Z. Thus, ab(ab − 2a) = 2mb(ab − 2a) which is even since mb(ab − 2a) is an integer.
Similarly, b even implies that b = 2n, ∈ Z. Hence, ab(ab−2a) = 2na(ab−2a) is even since na(ab−2a)
is an integer. The case where both a and b are even can be done similarly.

3. Suppoise x ∈ R. If x2 + 5x < 0 then x < 0.


Proof. We show that if x ≥ 0, then x2 + 5x ≥ 0. If x = 0, x2 + 5x = 02 + 5(0) = 0 ≥ 0. On the other
hand, if x > 0, then x + 5 > 0. This means that x2 + 5x = x(x + 5) is a product of two positive real
numbers. Thus, x2 + 5x > 0. In each case, x2 + 5x ≥ 0.

4. Suppose a, b ∈ Z. If both ab and a + b are even, then both a and b are even.

Proof. We show that if a or b is odd, then ab or a + b is odd. To prove this statement, we can either
assume ab is even and prove that a + b is odd or assume that a + b is even and prove that ab is odd.
We show that if a + b is even, then ab is odd. Note that the assumptions "a + b is even" and "a or b
is odd" forces a and b to be both odd. Without loss of generality, assume a = 2j + 1 and b = 2k + 1
for some integers j and k. Observe that ab = (2j + 1)(2k + 1) = 2(2kj + k + j) + 1. Since 2kj + k + j
is an integer, ab is an odd integer.

5. Suppose x, y ∈ Z. If x2 (y + 3) is even, then x is even or y is odd.

Proof. We show that if x is odd and y is even, then x2 (y +3) is odd. Let j and k be integers such that
x = 2j+1 and y = 2k. By substitution, x2 = (2j+1)2 = 2(2j 2 +2j)+1 and y+3 = 2k+3 = 2(k+1)+1.
This shows that x2 is an odd number and y + 3 is an odd number. Hence, x2 (y + 3) is odd since it
is a product of two odd numbers (see item 4).

Proof by Contradiction

Exercises. Use the method of proof by contradiction to prove the following statements.

1. Suppose n ∈ Z. If n is odd, then n2 is odd.

Proof. We assume the premise is true and the conclusion is false. We will derive a contradiction.
Suppose n2 is even. Since n is odd, n = 2k+1 for some integer k. By substitution, n2 = 2(2k 2 +2k)+1
is odd. This contradicts the assumption that n2 is even. This contradiction shows that if n is odd,
then n2 is odd.

2. Prove that 3 is irrational.
√ √
Proof. Suppose 3 is rational. It follows that there exists integers a, b ∈ Z such that 3 = ab with
2
b 6= 0 and gcd(a, b) = 1. From here we see that ab2 = 3 or a2 = 3b2 . The left hand side is equal to
a multiple of 3 which means that it is also a multiple of 3. Since the left hand side expression is a
square, 32 must be a divisor of a2 . Let a = 32 k 2 = (3k)2 , k ∈ Z. Thus, a = 3k and

a2 =3b2
32 k 2 =3b2
3k 2 =b2 .

Thus, b2 = 3k 2 . Observe that since b2 is equal to a multiple of 3, b2 = 32 j 2 = (3j)2 . Hence, b = 3j


and
√ so 3 is a divisor of both a and b. This contradicts the assumption that gcd(a, b) = 1. Therefore,
3 is irrational.

3. If a, b ∈ Z, then a2 − 4b − 3 6= 0.

Proof. Suppose to the contrary that there exists integers a and b such that a2 − 4b − 3 = 0. If follows
that a2 = 4b + 3 = 4(b + 43 ). Since a2 is an integer and a perfect square and 4 is a perfect square,
(b + 34 ) is a perfect square. But b integer implies (b + 43 ) is not an integer contradicting the fact that
a2 is an integer. This contradiction shows that if a, b ∈ Z, then a2 − 4b − 3 6= 0.

4. Suppose a, b ∈ R. If a is rational and ab is irrational, then b is irrational.

Proof. We assume a, b is rational, and ab is irrational. These assumptions implies that there are
integers p, q, r, s with q, s 6= 0 and a = pq , b = rs . Observe that ab = pq · rs = pr
qs
. Note that pr and qs
are integers since p, q, r, and s are integers. Moreover, qs 6= 0 since q, s 6= 0. Thus, ab is a rational
contradicting our assumption that it is irrational.

2
5. There exist no integers a and b for which 18a + 6b = 1.

Proof. Suppose there exists integers a and b such that 18a + 6b = 1. It follows that 18a + 6b =
2(9a + 3b) = 1. Since a and b are integers, 9a + 3b is an integer. Thus, 9a + 3b = 12 which contradicts
the fact that 9a + 3b is an integer.

If and only if Proof

Exercises. Prove the following statements.

1. Suppose x ∈ Z. Then x is even if and only if 3x + 5 is odd.

Proof. Let x ∈ Z. We first assume that x is even and show that 3x + 5 is odd. If x is even, then
x = 2k, k ∈ Z. By substitution, 3x + 5 = 3(2k) + 5 = 2(3k + 2) + 1. Since j = 3k + 2 is an integer,
3x + 5 = 2j + 1 is odd.
We next show that if 3x+5 is odd, then x is even. To prove this statement, we prove its contrapositive.
That is, we show that if x is odd, then 3x + 5 is even. If x is odd, then x = 2z + 1 for some integer
z. By substitution, 3x + 5 = 3(2z + 1) + 5 = 6z + 8 = 2(3z + 4). Since k = 3z + 4 is an integer,
3x + 5 = 2k is even.

2. Given an integer a, then a3 + a2 + a is even if and only if a is even.

Proof. Suppose a ∈ Z. We first show that if a3 + a2 + a is even, then a is even. To prove this, we
prove its contrapositive. That is, we show that if a is odd, then a3 + a2 + a is odd. If a is odd, then
a = 2r + 1, r ∈ Z. By substitution,

a3 + a2 + a =(2r + 1)3 + (2r + 1)2 + (2r + 1)


=8r3 + 16r2 + 12r + 3
=2(4r3 + 8r2 + 6r + 1) + 1.

We now see that a3 + a2 + a is odd.


We now show that if a is even, then a3 + a2 + a is even. The assumption a is even means that there
is an integer i such that a = 2i. Thus, a3 + a2 + a = a(a2 + a + 1) = 2i(a2 + a + 1). Since a and i
are integers, j = i(a2 + a + 1) is an integer. Hence, a3 + a2 + a = 2j is even.

3. An integer a is odd if and only if a3 is odd.

Proof. We first show that if a is odd, then a3 is odd. Note that if a is odd, then a · a = a2 is odd.
Thus, a3 = a2 · a is a product of two odd numbers. Therefore, a3 is odd.
We now show that if a3 is odd, then a is odd. We prove this statement by contradiction. Suppose
a3 is odd and a is even. By definition, a = 2h, h ∈ Z. Thus, a3 = (2h)3 = 8h3 = 2(4h3 ). We now see
that a3 is even contradicting the assumption.

4. Suppose a, b ∈ Z. Prove that (a − 3)b2 is even if and only if a is odd or b is even.

Proof. Suppose a, b ∈ Z. We first show that if (a − 3)b2 is even, then a is odd or b is even. We will
prove the contrapositive of this statement. That is, we show that if a is even and b is odd, then
(a − 3)b2 is odd. If a is even, then a − 3 is odd while if b is odd, then b2 is odd. Thus, (a − 3)b2 is a
product two odd numbers which is odd.
We next show that if a is odd or b is even, then (a − 3)b2 is even. If a is odd, then a − 3 is even. Let
a − 3 = 2h, h ∈ Z. Thus, (a − 3)b2 = 2hb2 which is even.

5. Suppose a, b ∈ Z. Prove that a + b is even if and only if a and b have the same parity.

3
Proof. We first show that if a + b is even, then a and b have the same parity. We prove this statement
by contradiction. Suppose a and b have different parity. Without loss of generality assume a is even
and b is odd. That is, a = 2f and b = 2g+1 for some f, g ∈ Z. Thus, a+b = 2f +2g+1 = 2(f +g)+1.
This contradicts the assumption that a + b is even. Therefore, if a + b is even, then a and b have the
same parity.
We next show that if a and b have the same parity, then a + b is even. Note that if a and b are both
even, a = 2r and b = 2s for some positive integers r and s. By substitution, a + b = 2r + 2s = 2(r + s)
which is even. If a and b are both odd, then a = 2u + 1 and b = 2v + 1 for some integers u and v.
By substitution, a + b = 2u + 1 + 2v + 1 = 2(u + v + 1) which is even. In any case, if a and b have
the same parity, then a + b is even.

Proving Two Sets Equal

Exercises. Prove the following statements.

1. If A, B and C are sets, then A ∩ (B ∪ C) = (A ∩ B) ∪ (A ∩ C).

Proof. We first show that A ∩ (B ∪ C) ⊆ (A ∩ B) ∪ (A ∩ C). We can accomplish this by showing that
an element in A ∩ (B ∪ C) also in (A ∩ B) ∪ (A ∩ C). Let x ∈ A ∩ (B ∪ C). It follows that x ∈ A
and x ∈ (B ∪ C). If x ∈ (B ∪ C), then x ∈ B or C. This means that x ∈ A and x ∈ B or x ∈ A and
x ∈ C. In symbols, x ∈ (A ∩ B) ∪ (A ∩ C).
We next show that A ∩ (B ∪ C) ⊇ (A ∩ B) ∪ (A ∩ C). We can. accomplish this by showing that
an element in (A ∩ B) ∪ (A ∩ C) also in A ∩ (B ∪ C). Let y ∈ (A ∩ B) ∪ (A ∩ C). It follows that
y ∈ (A ∩ B) or y ∈ (A ∩ C). That is, y is in A and B or y is in A and C. Therefore, y is in A and y
is in B or C. In symbols, this means that y ∈ A ∩ (B ∪ C).
Therefore, A ∩ (B ∪ C) = (A ∩ B) ∪ (A ∩ C).

2. If A and B are sets in a universal set U , then (A ∪ B)c = Ac ∩ B c .

Proof. We first show that (A ∪ B)c ⊆ Ac ∩ B c . Let x ∈ (A ∪ B)c . It follows that x ∈


/ (A ∪ B). That
is, x ∈ / B. We can now say that x ∈ A and x ∈ B . Therefore, x ∈ A ∩ B c .
/ A and x ∈ c c c

We next prove that (A ∪ B)c ⊇ Ac ∩ B c . Let y ∈ Ac ∩ B c . It follows that y ∈ Ac and y ∈ B c . This


means that y ∈
/ A and y ∈
/ B. That is, y ∈/ A ∪ B. Hence, y ∈ (A ∪ B)c .

3. If A and B are sets in a universal set U , then (A ∩ B)c = Ac ∪ B c .

Proof. The proof is similar to (2). We only prove (A ∩ B)c ⊆ Ac ∪ B c . Let x ∈ (A ∩ B)c . It follows
that x ∈
/ (A ∩ B). That is, x ∈ / B. Thus, x ∈ Ac or x ∈ B c . Hence x ∈ Ac ∪ B c . Tracing
/ A or x ∈
the arguments backwards can lead us to (A ∩ B)c ⊇ Ac ∪ B c . We leave it to the reader to write its
proof.

4. If A, B and C are sets, then A ∩ (B − C) = (A ∩ B) − (A ∩ C).

Proof. For variety, we manipulate the LHS of the equation and show that it is equal to the RHS of
the equation. Observe that

A ∩ (B − C) ={x : x ∈ A ∩ (B − C)}
={x : x ∈ A and x ∈ (B − C)}
={x : x ∈ A, x ∈ B, and x ∈
/ C}
={x : x ∈ (A ∩ B) − (A ∩ C)}
=(A ∩ B) − (A ∩ C).

Therefore, A ∩ (B − C) = (A ∩ B) − (A ∩ C).

5. If A, B and C are sets, then A − (B ∪ C) = (A − B) ∩ (A − C).

4
Proof. We prove as in (4). Note that
A − (B ∪ C) ={x : x ∈ A − (B ∪ C)}
={x : x ∈ A and x ∈/ (B ∪ C)}
={x : x ∈ A, x ∈
/ B, x ∈
/ C}
={x : x ∈ (A − B) and x ∈ (A − C)}
={x : x ∈ (A − B) ∩ (A − C)}
=(A − B) ∩ (A − C).

Mathematical Induction

Exercises. Prove the following statements.


n(n + 1)
1. For every integer n ∈ N, show that 1 + 2 + 3 + · · · + n = .
2
n(n + 1)
Proof. Let n ∈ N and Pn be the statement that 1 + 2 + 3 + · · · + n = . Clearly, P1 is true
2
1(1 + 1)
since 1 = . We next show that if k is a positive integer and Pk is true, then Pk+1 is also true.
2
k(k + 1)
If Pk is true, then 1 + 2 + 3 + · · · + k = . Observe that
2
k(k + 1)
1 + 2 + 3 + ··· + k =
2
k(k + 1)
(1 + 2 + 3 + · · · + k) + (k + 1) = + (k + 1)
2
(k 2 + k) + (2k + 2)
=
2
k 2 + 3k + 2
=
2
(k + 1)[(k + 1) + 1]
= .
2
(k + 1)[(k + 1) + 1]
Thus, 1 + 2 + 3 + · · · + k + (k + 1) = . That is, Pk+1 is true. By the principle of
2
Mathematical Induction, Pn is true for all positive integers n.
n(n + 1)(2n + 1)
2. For every integer n ∈ N, show that 12 + 22 + 32 + · · · + n2 = .
6
n(n + 1)(2n + 1)
Proof. Let n ∈ N and Pn be the statement that 12 + 22 + 32 + · · · + n2 = . We first
6
1(1 + 1)(2 · 1 + 1)
note that P1 is true since 12 = 1 = . We next show that if k is a positive integer
6
k(k + 1)(2k + 1)
and Pk is true, then Pk+1 is also true. If Pk is true, then 12 + 22 + 32 + · · · + k 2 = .
6
Observe that
k(k + 1)(2k + 1)
12 + 22 + 32 + · · · + k 2 =
6
k(k + 1)(2k + 1)
(12 + 22 + 32 + · · · + k 2 ) + (k + 1)2 = + (k + 1)2
6
k(k + 1)(2k + 1) + 6(k + 1)2
=
6
(k + 1)[k(2k + 1) + 6(k + 1)]
=
6
2
(k + 1)[2k + 7k + 6]
=
6
(k + 1)(k + 2)(2k + 3)
=
6
(k + 1)[(k + 1) + 1][2(k + 1) + 1)]
= .
6

5
(k + 1)[(k + 1) + 1][2(k + 1) + 1)]
Thus, 12 + 22 + 32 + · · · + k 2 + (k + 1)2 = . That is, Pk+1 is true.
6
By the principle of Mathematical Induction, Pn is true for all positive integers n.

n2 (n + 1)2
3. For every integer n ∈ N, show that 13 + 23 + 33 + · · · + n3 = .
4

n2 (n + 1)2
Proof. Let n ∈ N and Pn be the statement that 13 + 23 + 33 + · · · + n3 = . We first note
4
12 (1 + 1)2
that P1 is true since 13 = 1 = . We next show that if k is a a positive integer and Pk is
4
k 2 (k + 1)2
true, then Pk+1 is also true. If Pk is true, then 13 + 23 + 33 + · · · + k 3 = . Observe that
4
k 2 (k + 1)2
13 + 23 + 33 + · · · + k 3 =
4
2
3 3 3 3 3 k (k + 1)2
(1 + 2 + 3 + · · · + k ) + (k + 1) = + (k + 1)3
4
k 2 (k + 1)2 + 4(k + 1)3
=
4
(k + 1)2 [k 2 + 4(k + 1)]
=
4
2 2
(k + 1) [k + 4k + 4)]
=
4
(k + 1)2 (k + 2)2
=
4
(k + 1) [(k + 1) + 1]2
2
= .
4
(k + 1)2 [(k + 1) + 1]2
Thus, 13 + 23 + 33 + · · · + k 3 + (k + 1)3 = . That is, Pk+1 is true. By the principle
4
of Mathematical Induction, Pn is true for all positive integers n.

n(n + 1)(n + 2)
4. For every integer n ∈ N, show that 1(2) + 2(3) + 3(4) + · · · + n(n + 1) = .
3

n(n + 1)(n + 2)
Proof. Let n ∈ N and Pn be the statement that 1(2) + 2(3) + 3(4) + · · · + n(n + 1) = .
3
1(1 + 1)(1 + 2)
Clearly, P1 is true since 1(2) = 2 = . We next show that if k is a positive integer and
3
k(k + 1)(k + 2)
Pk is true, then Pk+1 is also true. If Pk is true, then 1(2)+2(3)+3(4)+· · ·+k(k+1) = .
3
Observe that
k(k + 1)(k + 2)
1(2) + 2(3) + 3(4) + · · · + k(k + 1) =
3
k(k + 1)(k + 2)
(1(2) + 2(3) + 3(4) + · · · + k(k + 1)) + (k + 1)(k + 2) = + (k + 1)(k + 2)
3
k(k + 1)(k + 2) + 3(k + 1)(k + 2)
=
3
(k + 1)(k + 2)(k + 3)
=
3
(k + 1)[(k + 1) + 1][(k + 1) + 2]
= .
3
(k + 1)[(k + 1) + 1][(k + 1) + 2]
Thus, 1(2) + 2(3) + 3(4) + · · · + k(k + 1) = . That is, Pk+1 is true.
3
By the principle of Mathematical Induction, Pn is true for all positive integers n.

5. Prove that for all n ≥ 1,

(an − 1) = (a − 1)(an−1 + an−2 + an−3 + · · · + a + 1).

6
Proof. Let n be a positive integer and Pn be the statement that (an − 1) = (a − 1)(an−1 + an−2 +
an−3 + · · · + a + 1). Clearly, P1 is true since (a1 − 1) = a − 1 = (a − 1)(1). We next show that
if k is a positive integer and Pk is true, then Pk+1 is also true. If Pk is true, then (ak − 1) =
(a − 1)(ak−1 + ak−2 + ak−3 + · · · + a + 1). Observe that
ak+1 − 1 =ak+1 − a + a − 1
=a(ak − 1) + (a − 1)
=a[(a − 1)(ak−1 + ak−2 + ak−3 + · · · + a + 1)] + (a − 1)
=(a − 1)[a(ak−1 + ak−2 + ak−3 + · · · + a + 1) + 1]
=(a − 1)(ak + ak−1 + ak−2 + ak−3 + · · · + a + 1)
=(a − 1)(a(k+1)−1 + a(k+1)−2 + a(k+1)−3 + · · · + a + 1).
Thus, ak+1 − 1 = (a − 1)(a(k+1)−1 + a(k+1)−2 + a(k+1)−3 + · · · + a + 1). That is, Pk+1 is true. By the
principle of Mathematical Induction, Pn is true for all positive integers n.

6. If the number an are defined inductively by a1 = 11, a2 = 21, and an = 3an−1 − 2an−2 for all n ≥ 3.
Prove that an = 5 · 2n + 1 for every positive integer n.

Proof. We prove this using strong induction. Let n be a positive integer and Pn be the statement
that an = 5 · 2n + 1. We first verify whether P1 , P2 , and P3 are true.
If n = 1, then a1 = 11 = 5 · 21 + 1.
If n = 2, then a2 = 21 = 5 · 22 + 1.
If n = 3, then a3 = 3a2 − 2a1 = 3(21) − 2(11) = 41 = 5 · 23 + 1.
Thus, P1 , P2 , and P3 are all true. We next show that if m and k are integers and Pm is true for all
3 ≤ m ≤ k, then Pk+1 is also true. If Pm is true, then am = 5 · 2m + 1 for all 3 ≤ m ≤ k. Observe
that
ak+1 =3ak − 2ak−1
=3(5 · 2k + 1) − 2(5 · 2k−1 + 1)
=15 · 2k + 3 − 5 · 2k − 2
=10 · 2k + 1
=5 · 2k+1 + 1.
Thus, ak+1 = 5 · 2k+1 + 1. That is, Pk+1 is true. By the principle of Mathematical Induction(strong
form), Pn is true for all positive integers n.
n
3
3i = (3n − 1).
X
7. Prove that:
i=1 2
n
3 n
3i =
X
Proof. Let n be a positive integer and Pn be the statement that (3 − 1). Clearly, P1 is
i=1 2
1
3
3i = 3 = (31 − 1). We next show that if k is a positive integer and Pk is true, then
X
true since
i=1 2
k
3
3i = (3k − 1). Observe that
X
Pk+1 is also true. If Pk is true, then
i=1 2
k
3
3i = (3k − 1)
X

i=1 2
k
!
i 3
+ 3k+1 = (3k − 1) + 3k+1
X
3
i=1 2
k+1
3k+1 − 3 + 2 · 3k+1
3i =
X

i=1 2
3 · 3k+1 − 3
=
2
3 k+1
= (3 − 1).
2

7
k+1
3
3i = (3k+1 − 1). That is, Pk+1 is true. By the principle of Mathematical Induction, Pn is
X
Thus,
i=1 2
true for all positive integers n.

8. Prove that n < 2n for any positive integer n.

Proof. Let n be a positive integer and Pn be the statement that n < 2n . Clearly, P1 is true since
1 < 21 . We next show that if k is a positive integer and Pk is true, then Pk+1 is also true. If Pk is
true, then k < 2k . Observe that

k + 1 ≤ k + k = 2k < 2 · 2k = 2k+1 .

Thus, k + 1 < 2k+1 . That is, Pk+1 is true. By the principle of Mathematical Induction, Pn is true for
all positive integers n.

9. Prove that (n + 1)n < nn+1 for any integer n ≥ 3.

Proof. Let n ≥ 3 be a positive integer and Pn be the statement that (n + 1)n < nn+1 . Clearly, P3 is
true since 64 = (3 + 1)3 < 33+1 = 81. We next show that if k ≥ 3 is a positive integer and Pk is true,
then Pk+1 is also true. If Pk is true, then (k + 1)k < k k+1 . Observe that

(k + 1)k <k k+1


(k + 2)k+1 (k + 1)k <(k + 2)k+1 k k+1 = [(k + 2)k]k+1 = (k 2 + 2k)k+1 .

Thus, (k + 2)k+1 (k + 1)k < (k 2 + 2k)k+1 < (k 2 + 2k + 1)k+1 = (k + 1)2k+2 = (k + 1)k+2 (k + 1)k .
Hence, (k + 2)k+1 (k + 1)k < (k + 1)k+2 (k + 1)k or (k + 2)k+1 < (k + 1)k+2 .
This shows that Pk+1 is true. By the principle of Mathematical Induction, Pn is true for all positive
integers n ≥ 3.

10. Prove that n2 + 7n is divisible by 2 for any positive integer n.

Proof. Let n be a positive integer and Pn be the statement that n2 + 7n is divisible by 2. Clearly,
P1 is true since 12 + 7(1) = 8 is divisible by 2. We next show that if k is a positive integer and Pk is
true, then Pk+1 is also true. If Pk is true, then k 2 + 7k is divisible by 2. Observe that

(k + 1)2 + 7(k + 1) =(k + 1)[(k + 1) + 7]


=(k + 1)(k + 8)
=k 2 + 9k + 8
=k 2 + 7k + 2k + 8
=(k 2 + 7k) + 2(k + 4).

Since k 2 + 7k is divisible by 2, it is even. On the other hand, since k is an integer, 2(k + 4) is even.
Thus, (k + 1)2 + 7(k + 1) = (k 2 + 7k) + 2(k + 4) is even since it is a sum of two even integers. Hence,
(k + 1)2 + 7(k + 1) is divisible by 2. This shows that Pk+1 is true. By the principle of Mathematical
Induction, Pn is true for all positive integers n.

11. Prove that 10n+1 + 3 · 10n + 5 is divisible by 9 for any positive integer n.

Proof. Let n be a positive integer and Pn be the statement that 10n+1 + 3 · 10n + 5 is divisible by 9.
Clearly, P1 is true since 101+1 + 3 · 101 + 5 = 135 = 9(15) is divisible by 9. We next show that if k
is a positive integer and Pk is true, then Pk+1 is also true. If Pk is true, then 10k+1 + 3 · 10k + 5 is
divisible by 9. Observe that

10(k+1)+1 + 3 · 10k+1 + 5 =10 · 10k+1 + 3 · 10k · 10 + 5


=10(10k+1 + 3 · 10k ) + 5
=10(10k+1 + 3 · 10k + 5) + 5 − 50
=10(10k+1 + 3 · 10k + 5) − 45.

8
By the inductive assumption, 10k+1 + 3 · 10k + 5 is divisible by 9. This means that there exists an
integer j such that 10k+1 + 3 · 10k + 5 = 9j. By substitution,

10(k+1)+1 + 3 · 10k+1 + 5 =10(10k+1 + 3 · 10k + 5) − 45


=10(9j) − 45
=9(10j − 5).

Thus, 10(k+1)+1 + 3 · 10k+1 + 5 = 9(10j − 5) is divisible by 9. Hence, Pk+1 is true and by the principle
of Mathematical Induction, Pn is true for all positive integers n.

12. Prove that (x − y) is a factor of xn − y n .

Proof. Let n be a positive integer and Pn be the statement that (x − y) is a factor of xn − y n . Clearly,
P1 is true since (x − y) is a factor of x1 − y 1 = x − y. We next show that if k is a positive integer
and Pk is true, then Pk+1 is also true. If Pk is true, then (x − y) is a factor of xk − y k . Observe that

xk+1 − y k+1 =xk+1 − xy k + xy k − y k+1


=x(xk − y k ) + y k (x − y).

By the inductive assumption, (x − y) is factor of (xk − y k ). On the other hand, (x − y) is a factor of


y k (x − y). Hence, (x − y) is a factor of xk+1 − y k+1 = x(xk − y k ) + y k (x − y). That is, Pk+1 is true.
By the principle of Mathematical Induction, Pn is true for all positive integers n.

13. Brain Boggler at Discover Magazine, 1988. The Quakertown Poker Club plays with blue chips
worth $5.00 and red chips worth $8.00. What is the largest bet that cannot be made?
Claim: The largest bet that cannot be made is $27. That is, we can bet n dollars with $5 blue chips
and $8 red chips for all positive integers n ≥ 28. We will use strong induction to prove this claim.

Proof. Let n ≥ 28 be a positive integer and Pn be the statement that we can bet n dollars with
$5 blue chips and $8 red chips. That is, Pn is the statement that n = 5a + 8b where a and b are
nonnegative numbers with a and b not both zero. Clearly, P28 is true since 28 = 5(4) + 8(1). We
next show that if m and k are positive integers and Pm is true for all 28 ≤ m ≤ k, then Pk+1 is also
true. If Pm is true for all 28 ≤ m ≤ k, then m = 5r + 8s for some nonnegative integers r and s. Note
that since 28 ≤ m ≤ k, either r or s is atleast 3. If r ≥ 3, then

k + 1 =5r + 8s + 1
=5r + 8s + [5(−3) + 8(2)]
=5(r − 3) + 8(s + 2).

This means that if r blue chips and s red chips is k dollars then r − 3 blue chips and s + 2 chips is
k + 1 dollars. Since r ≥ 3, this is a valid betting option.
Similarly, if s ≥ 3, then

k + 1 =5r + 8s + 1
=5r + 8s + [5(5) − 8(3)]
=5(r + 5) + 8(s − 3).

This means that r + 5 blue chips and s − 3 chips is k + 1 dollars. Since s ≥ 3, this is a valid betting
option. Thus, k + 1 = 5j + 8k. for some nonnegative integers j and k. That is, Pk+1 is true. By the
principle of Mathematical Induction(strong form), Pn is true for all positive integers n ≥ 28.

Additional Exercises. Prove the following statements.

1. Show that there is no real number x which satisties x2 + 4 = 0.

Proof. We prove this statement by contradiction. Suppose there exists a real number x such that
x2 + 4 = 0. This means that x2 = −4. That is, there is a real number x whose square is negative.
This contradicts the fact that the square of a real number is either zero or positive. Therefore, there
is no real number x which satisties x2 + 4 = 0.

9
2. For every nonnegative integer n, prove that a set with exactly n elements has exactly 2n subsets.

Proof. We prove this using Mathematical Induction. Let n be a nonnegative integer and Pn be the
statement that a set with n elements has 2n subsets.
Clearly, P0 is true since the empty set has 20 = 1 subset, which is itself. On the other hand, P1 is
true since a set with one element has 21 = 2 subsets, the empty set and itself.
Let k be a nonnegative integer and S = {a1 , a2 , . . . , ak } be a set with k elements. We assume that
S has 2k elements. That is, we assume Pk is true. We want to show that Pk+1 is true. That is, we
show that the set T = S ∪ {ak+1 } with k + 1 elements has 2k+1 subsets.
By assumption, S has 2k subsets, let S1 , S2 , . . . , S2k be the distinct subsets of S. Observe that all
subsets of S are also subsets of T . Moreover, S1 ∪ {ak+1 }, S2 ∪ {ak+1 }, . . . , S2k ∪ {ak+1 } are also
distinct subsets of T that are not subsets of S. Thus,

S1 , S2 , . . . , S2k , S1 ∪ {ak+1 }, S2 ∪ {ak+1 }, . . . , S2k ∪ {ak+1 }

is the complete list of subsets of T . Counting this list shows that T has 2k + 2k = 2 · 2k = 2k+1
subsets. Therefore, Pk+1 is true and so every sets with n elements must have 2n subsets.

3. Show that every integer greater than 1 is a prime or a product of primes.

Proof. We prove this statement using the strong form of Mathematical Induction. Let n > 1 be an
integer and Pn be the statement that n is a prime or a product of primes.
Note that P2 , P3 , and P4 are all true since 2 and 3 are primes and 4 = 22 is a product of primes.
Let m and k be integers with 4 ≤ m ≤ k. Suppose Pm is true. We want to show that Pk+1 is also
true. If Pm is true, then m is a prime or a product of primes for all 4 ≤ m ≤ k.
Consider k +1. If k +1 is prime, then we are done. Suppose k +1 is composite. Then k +1 = ab where
a and b are positive integers with a, b < k + 1. It follows that a, b ≤ k. By the inductive assumption,
a and b are primes or product of primes. Thus, k + 1 = ab is a product of primes. Therefore, Pk+1
is true and so by strong induction, every integer greater than 1 is a prime or a product of primes.

4. Let p1 , p2 , . . . , pn be primes. Show that p1 p2 · · · pn + 1 is divisible by none of these primes. Use this
to prove that there are infinitely many primes.

Proof. Let n be a positive integer and p1 , p2 , . . . , pn be primes. We want to show that p1 p2 · · · pn + 1


is not divisible by pi for all i = 1, 2, . . . , n. Our proof is by contradiction. Suppose there exists
1 ≤ j ≤ n such that pj divides p1 p2 · · · pn + 1. It follows that there exist an integer m such that
p1 p2 · · · pn + 1 = pj m. Thus, 1 = pj m − p1 p2 · · · pn = pj (m − p1 p2 · · · pj−1 pj+1 · · · pn ). Since m is an
integer, m − p1 p2 · · · pj−1 pj+1 · · · pn = q is an integer. Hence, 1 = pj q. Since pj is a prime, this last
equation can only be true if q = p1j which contradicts the fact that q is an integer. This contradiction
shows that p1 p2 · · · pn + 1 is not divisible by pi for all i = 1, 2, . . . , n.
We now prove that there are infinitely many primes. Suppose to the contrary that there are a finite
number of primes. Let p1 , p2 , · · · , pn be the list of all primes. From above, p1 p2 · · · pn + 1 is not
divisible by any of the primes p1 , p2 , · · · , pn . This means that p1 p2 · · · pn + 1 is also a prime. Since
p1 p2 · · · pn + 1 is larger than any of the primes p1 , . . . , pn , p1 p2 · · · pn + 1 is a prime that is not included
in the original list. This contradicts the fact that there are only n primes.

5. Remainder Theorem. If a polynomial P (x) is divided by x − c, then the remainder is P (c).

Proof. The Division Algorithm guarantees that if P (x) is divided by x − c then there exists unique
polynomials Q(x) and R such that P (x) = Q(x)(x − c) + R and R is a constant called the remainder.
Observe that if x = c, then P (c) = Q(c)(c − c) + R = R. Thus, the remainder when P (x) is divided
by (x − c) is P (c) = R.

6. Factor Theorem. A polynomial P (x) has a factor x − c if and only if P (c) = 0.

10
Proof. We first show that if P (x) has factor (x − c), then P (c) = 0. If P (x) has factor (x − c), then
the remainder is zero when P (x) is divided by (x − c). By the Remainder theorem, P (c) = 0.
We next show that if P (c) = 0, then P (x) has a factor (x − c). Suppose P (c) = 0. By the Division
Algorithm, there are polynomials Q(x) and R such that P (x) = Q(x)(x−c)+R. From the Remainder
Theorem and our assumption, R = P (c) = 0. Thus, P (x) = Q(x)(x − c) + R = Q(x)(x − c). This
shows that P (x) = Q(x)(x − c). That is, (x − c) is a factor of P (x).

7. Rational Zeros Theorem. If the polynomial function

P (x) = an xn + an−1 xn−1 + an−2 xn−2 + · · · + a1 x + a0

has integer coefficients, then every rational zero of P (x) is of the form pq (in lowest terms), where p
is a factor of the constant coefficient a0 and q is a factor of the leading coefficient an .

Proof. Let P (x) = an xn + an−1 xn−1 + an−2 xn−2 + · · · + a1 x + a0 be a polynomial function with
integer coefficients. We want to show that if pq (in lowest terms) is a rational zero of P (x) =
an xn + an−1 xn−1 + an−2 xn−2 + · · · + a1 x + a0 , then p divides a0 and q divides an . If pq (in lowest terms)
is a rational zero of P (x), then

P ( pq ) = an ( pq )n + an−1 ( pq )n−1 + an−2 ( pq )n−2 + · · · + a1 ( pq ) + a0 = 0.

Thus,

an ( pq )n + an−1 ( pq )n−1 + an−2 ( pq )n−2 + · · · + a1 ( pq ) + a0 =0


an ( pq )n + an−1 ( pq )n−1 + an−2 ( pq )n−2 + · · · + a1 ( pq ) = − a0
(an ( pq )n + an−1 ( pq )n−1 + an−2 ( pq )n−2 + · · · + a1 ( pq ))q n = − a0 q n
an pn + an−1 pn−1 q + an−2 pn−2 q 2 + · · · + a1 pq n−1 = − a0 q n (†)
p(an pn−1 + an−1 pn−2 q + an−2 pn−3 q 2 + · · · + a1 q n−1 ) = − a0 q n . (?)

Notice that the LHS of Equation (?) is divisible by p. Thus, the RHS, −a0 q n is divisible by p. Since
p and q are relatively prime, p cannot divide q n . Hence, p must be a divisor of a0 .
On the other hand, Equation (†) implies that

q(an−1 pn−1 + an−2 pn−2 q + · · · + a1 pq n−2 + a0 q n−1 ) = −an pn . (∗)

Since RHS of Equation (∗) is divisible by q, −an pn is also divisible by q. Using the same argument
above, we see that an is divisible by q. This completes the proof of the Rational Zeros Theorem.

8. Prove that 5 − 1 is irrational.
√ √
Proof. We use the Rational Zeros Theorem to prove that 5 − 1 is irrational. Let x = 5 − 1. Then

x= 5−1

x+1= 5
(x + 1)2 =5
x2 + 2x + 1 =5
x2 + 2x − 4 =0.

The Rational Zeros theorem


√ tells us that the only possible
√ rational zeros of P (x) = x2 + 2x
√ − 4 are
±1, ±2, ±4. Since x =√ 5 − 1 is a zero of P (x) and 5 − 1 ∈ / {±1, ±2, ±4}, we see that 5 − 1 is
not rational. That is, 5 − 1 is irrational.

x2 − 1
9. Show by contradiction that f (x) = has no inverse.
x
Proof. A function g is the inverse of f if the following conditions are satisfied.

11
(i) For all x in the domain of f , f (x) is in the domain of g and g(f (x)) = x.
(ii) For all x in the domain of g, g(x) is in the domain of f and f (g(x)) = x.
x2 − 1
We show that if f (x) = , then no such g satisfy the above conditions. Suppose g is an inverse
x
(±1)2 − 1
of f satisfying the conditions given above. If x = ±1, then f (±1) = = 0. Thus,
±1
−1 = g(f (−1)) = g(0) = g(f (1)) = 1.

This contradicts the fact that −1 6= 1. Thus, f has no inverse.

10. A function f is said to be one-to-one function if for any two elements x1 , x2 in the domain of f such
1 − 2x
f (x1 ) = f (x2 ) in the range of f , then x1 = x2 . Prove that f (x) = is a one-to-one function.
x+3

Proof. We assume f (x1 ) = f (x2 ) in the range of f and show that x1 = x2 in the domain of f . If
f (x1 ) = f (x2 ), then x1 , x2 are in the domain of f and
1 − 2x1 1 − 2x2
=
x1 + 3 x2 + 3
(1 − 2x1 )(x2 + 3) =(1 − 2x2 )(x1 + 3)
x2 − 2x1 x2 + 3 − 6x1 =x1 − 2x2 x1 + 3 − 6x2
7x2 =7x1
x2 =x1 .

Since x1 = x2 in the domain of f , f is 1-1.

11. Suppose that P (x) = an xn + an−1 xn−1 + an−2 xn−2 + · · · + a1 x + a0 is a polynomial function and that
an > 0. In the synthetic division of P (x) by x − r,
(i) If r > 0 and the numbers in the third row are all nonnegative numbers, then r is an upper bound
for the real zeros of P .
(ii) If r < 0 and the numbers in the third row are alternately nonnegative and non-positive, then r
is an lower bound for the real zeros of P .

Proof. Assume the conditions of the theorem. We first show that if r > 0 and the numbers in the
third row are all nonnegative numbers in the synthetic division of P (x) by x − r, then r is an upper
bound for the real zeros of P . Recall that r is an upper bound for the real zeros of P if every zero
of P is at most r. That is, r is an upper bound for the real zeros of P if there exists no real number
s > r > 0 for which P (s) = 0.
Before proving (i), we note that by the Division Algorithm, we can write P (x) = Q(x)(x − r) + R
where Q(x) is a polynomial whose degree is one less than that of P (x) and R is a constant.
Let s > r > 0. We will show that P (s) 6= 0. Observe that the conditions in (i) shows that
Q(s) > 0 and R ≥ 0. Also, since s > r, s − r > 0. Thus, P (s) = Q(s)(s − r) + R > 0. That is,
P (s) = Q(s)(s − r) + R 6= 0. Therefore, no other zeros of P are larger than r. This shows that r is
an upper bound for the real zeros of P .
To prove (ii), we assume the conditions of the theorem in (ii). Our goal is to show that if s < r < 0,
then P (s) 6= 0. That is, we show that no other zero of P is smaller than r. We consider two cases
based on the degree n of P .
Case 1. n is even.
In this case, the degree of Q(x) is n − 1 which is odd and Q(x) has at most n − 1 terms. Because
of the conditions in (ii), it is not difficult to see that Q(s) < 0, R ≥ 0, and s − r < 0. Thus,
P (s) = Q(s)(s − r) + R > 0. That is, P (s) = Q(s)(s − r) + R 6= 0. Therefore, no other zeros of P
are smaller than r.
Case 2. n is odd.
In this case, the degree of Q(x) is n − 1 which is even and Q(x) has at most n − 1 terms. Because
of the conditions in (ii), it is not difficult to see that Q(s) > 0, R ≤ 0, and s − r < 0. Thus,

12
P (s) = Q(s)(s − r) + R < 0. That is, P (s) = Q(s)(s − r) + R 6= 0. Therefore, no other zeros of P
are smaller than r.
In any case, we see that r is a lower bound for the real zeros of P .

12. Let h be a function defined on all real numbers such that for all real numbers x1 , x2 , h(x1 + x2 ) =
h(x1 ) + h(x2 ). Prove that h(nx) = n · h(x).

Proof. Suppose h(x1 + x2 ) = h(x1 ) + h(x2 ) for all real numbers x1 and x2 . We show that h(nx) =
n · h(x). Observe that

h(nx) =h([n − 1]x + x)


=h([n − 1]x) + h(x)
=h([n − 2]x + h(x) + h(x)
=h([n − 2]x) + h(x) + h(x)
=h([n − 3]x + x) + h(x) + h(x)
=h([n − 3]x) + h(x) + h(x) + h(x)
..
.
n-times
z }| {
= h(x) + h(x) + · · · h(x)
=n · h(x)

Therefore, h(nx) = n · h(x).

13. Let P (x1 , y1 ) q


and Q(x2 , y2 ) be two points in a plane. Prove that the distance d between P and Q is
given by d = (x2 − x1 )2 + (y2 − y1 )2 .

Proof. If P = Q, then the theorem is trivial. Suppose P 6= Q. We consider 3 cases.


Case 1. P and Q lie on a horizontal line.
In this case, y1 = y2 . Moreover, the distance d between P and Q is given by
q q
d = |x1 − x2 | = (x2 − x1 )2 = (x2 − x1 )2 + (y2 − y1 )2 .

Case 2. P and Q lie on a vertical line.


In this case, x1 = x2 . Moreover, the distance d between P and Q is given by
q q
d = |y1 − y2 | = (y2 − y1 )2 = (x2 − x1 )2 + (y2 − y1 )2 .

Case 3. P and Q lie on a nonvertical and nonhorizontal line.


In this case, we can form a right triangle P QS with hypotenuse is P Q. Without loss of generality,
assume P S is horizontal and QS is vertical. By Pythagorean theorem,

P Q2 =QS 2 + P S 2
=|y2 − y1 |2 + |x2 − x1 |2
=(y2 − y1 )2 + (x2 − x1 )2 .
q
Thus, P Q2 = (y2 − y1 )2 + (x2 − x1 )2 or P Q = (x2 − x1 )2 + (y2 − y1 )2 .

14. Let L1 and L2 be two distinct non-vertical lines with slopes m1 and m2 , respectively. Then L1 is
parallel to L2 if and only if m1 = m2 .

Proof. Let m1 , m2 , b1 , b2 be real numbers, y = m1 x+b1 be the equation of the line L1 , and y = mx2 +b2
be the equation of the line L2 .
We first assume that L1 k L2 and show that m1 = m2 . If L1 k L2 , then they have no point in
common. Thus, the equations y = m1 x + b1 and y = mx2 + b2 have no point in common. Note
that m1 x + b1 = y = mx2 + b2 . That is, x(m1 − m2 ) = m1 x − m2 x = b2 − b1 . Since the two lines

13
are distinct and no point in common, the two equations will have no solution when m1 − m2 = 0 or
m1 = m2 .
We next show that if m1 = m2 , then L1 k L2 . If m1 = m2 = m, then the equations of L1 and L2 can
be written as y = m1 x + b1 = mx + b1 and y = m2 x + b2 = mx + b2 , respectively. We claim that
these two lines have no point in common. To verify this claim, we suppose the two lines have a point
in common and derive a contradiction. Observe that mx + b1 = y = mx + b2 or b1 = b2 = b. This
means that the equation of the two lines is y = mx + b contradicting the fact that they are distinct
lines. Thus, the two lines have no point in common and so they are parallel.

15. Let L1 and L2 be two distinct non-vertical lines with slopes m1 and m2 , respectively. Then L1 is
perpendicular to L2 if and only if m1 m2 = −1.

Proof. We prove the statement for L1 and L2 intersecting at the origin. The proof of the general
case follows from translation of axes.
Suppose the equation of L1 is y = m1 x and the equation of L2 is y = m2 x.
We first show that if L1 ⊥ L2 , then m1 m2 = −1. We first construct a line x = 1 forming a right
triangle TAP with the lines y = m1 x and y = m2 x in the 1st and 4th quadrant on the Cartesian
plane (see figure).

Notice that the intersection of y = m1 x and x = 1qis T (1, m1 ) and q


the intersection of y = m2 x and
x = 1 is P (1, m2 ). By the distance formula, AT = 1 + m1 , AP = 12 + m22 , and P T = |m1 − m2 |.
2 2

By the Pythagorean Theorem, substitution, and by simplifying the expressions, we have

T P 2 =AT 2 + AP 2
q q
|m1 − m2 |2 =( 12 + m21 )2 + ( 12 + m22 )2
(m1 − m2 )2 =(1 + m21 ) + (1 + m22 )
m21 − 2m1 m2 + m22 =2 + m21 + m22
−2m1 m2 =2
m1 m2 = −1.

Thus, if two nonvertical lines are perpendicular, then the product of their slopes is −1.
We next assume that the product of the slopes of two nonvertical lines is −1. That is, if m1 is the
slope of L1 and m2 is the slope of L2 , then m1 m2 = −1. We want to show that L1 ⊥ L2 .
Let the equation of L1 be y = m1 x + b1 and the equation of L2 be y = m2 x + b2 = − m11 x + b2 .
We first show that L1 and L2 have an intersection point. Observe that m1 x + b1 = y = − m11 x + b2 .
m1 (b2 − b1 ) b2 − b1
Solving for x we have x = 2
. By substitution, y = 2 + b2 . Since m21 + 1 is a real
m1 + 1 m1 + 1

14
number for all m1 , we see that these values of x and y is a solution of the two equations. Thus, if
the product of the slopes of two distinct lines is −1, then they intersect.
Let us assume that the intersection of the lines L1 and L2 is the origin. The general case will follow
from translation of axes.
As before, we again construct a line x = 1 forming a right triangle TAP with the lines y = m1 x and
y = m2 x in the 1st and 4th quadrant on the Cartesian plane (see figure).

Notice that the intersection of y = m1 x and x = 1 is T (1, m1 ) and the intersection


s of y = m2 x
1 1 2
  q  
2
and x = 1 is P 1, − . By the distance formula, M T = 1 + m1 , T P = 1 + −
2 2 , and
m1 m1
1 m2 + 1
P T = |m1 + |= 1 . Observe that

m1 m1

2 2
s 
2
1
q 
2 2
TP + TM = 12 + m21 +  12 + − 
m1
!
1
=(1 + m21 ) + 1 + 2
m1
1
=2 + m21 + 2
m1
4 2
m + 2m1 + 1
= 1
m21
(m1 + 1)2
=
m21
!2
(m1 + 1)
=
m1
m2 + 1 2

1
= = P T 2.


m1

Thus, T P 2 + T M 2 = P T 2 . By the Converse of the Pythagorean Theorem, triangle M T P is a right


triangle. Thus, the lines L1 and L2 are perpendicular at the origin.
n
16. Use induction to prove that for every positive integer
√ n and every real number θ, (cos θ + i sin θ) =
cos nθ + i sin nθ, where i is the complex number −1.
n
Proof. Let n be a positive integer
√ and Pn be the statement that (cos θ + i sin θ) = cos nθ + i sin nθ,
where i is the complex number −1 and θ is a real number. Clearly, P1 is true since

(cos θ + i sin θ)1 = cos(1)θ + i sin(1)θ.

We next show that if k is a positive integer and Pk is true, then Pk+1 is also true. If Pk is true, then

(cos θ + i sin θ)k = cos kθ + i sin kθ.

15
By substitution,

(cos θ + i sin θ)k+1 =(cos θ + i sin θ)k (cos θ + i sin θ)


=(cos kθ + i sin kθ)(cos θ + i sin θ)
= cos θ cos kθ − sin θ sin kθ + i(sin kθ cos θ + cos kθ sin θ)
= cos(θ + kθ) + i sin(θ + kθ)
=. cos([k + 1]θ) + i sin([k + 1]θ).

This shows that Pk+1 is true. By the principle of Mathematical Induction, Pn is true for all positive
integers n.

17. In triangle ABC, BA is produced at D. If AE bisects ∠CAD at A and intersects BC produced at


E, prove that BE : CE = AB : AC.

Proof. We assume the conditions of the theorem. We want to prove that BE : CE = AB : AC.
We construct a segment P C parallel to AE and P is on BA (see figure).

We first note that since P C k AE, by the Basic Proportionality Theorem, BE : CE = AB : AP .


The theorem will be proven if we can show that AP = AC.
Since AE k P C, ∠DAE = ∠DP C since they are corresponding angles using BD as the transversal
line. On the other hand, using AC as the transversal, ∠EAC = ∠P CA since they are alternate
interior angles. Because EA bisects ∠DAC, ∠DAE = ∠EAC. By substitution, ∠DP C = ∠P CA.
Thus, triangle ADC is isosceles. This means that AP = AC.
By substitution, we have BE : CE = AB : AC.

18. Prove that the diagonals of a parallelogram bisect each other.

Proof. Let ABCD be a parallelogram with diagonals AC and BD intersecting at E (see figure).

Since ABCD is a parallelogram, AB = CD. On the other hand, ∠AEB = ∠DEC since they are
vertical angles. Finally, ∠1 = ∠2 since they are alternate interior angles with transversal BD. By
SAA, 4DEC ∼ = 4BEA. By CPCTE, AE = CE and BE = DE. Since AE + CE = AC and
BE + DE = BD, the diagonals of a parallelogram bisect each other.

16
19. Pythagorean Theorem In a right triangle, the square of the hypotenuse is equal to the sum of the
squares of the two legs.

Proof. Let 4ABC be a right triangle. Let AC be the hypotenuse and Y a point on AC such that
BY ⊥ AC (see figure).

By the Geometric Mean Theorem, AB 2 = AY (AC) and BC 2 = CY (AC). Thus,

AB 2 + BC 2 = AY (AC) + CY (AC) = (AY + CY )AC = AC(AC) = AC 2 .

Therefore, AB 2 + BC 2 = AC 2 . That is, in a right triangle, the square of the hypotenuse is equal to
the sum of the squares of the two legs.

20. An angle inscribed in a circle is half the central angle intercepting the same arc.

Proof. Let θ be the measure of an angle inscribed in a circle. Let ω be the measure of the central
angle intercepting the same arc intercepted by θ. We want to show that θ = 12 ω. We consider 3
cases.
Case 1. One side of angle θ is a diameter of the circle (see figure).

Notice that AD and AB are radii of the circle, so AD = AB. It follows that 4ABD is an isosceles
triangle. Thus, θ = α. Since ω is an exterior angle of 4ABD, ω = θ + α = θ + θ = 2θ. This shows
that θ = 12 ω.
Case 2. A diameter is between the sides of angle θ (see figure).
We let ω = ω1 + ω2 and θ = θ1 + θ2 . From Case 1, θ1 = 21 ω2 and θ2 = 12 ω1 . By Substitution,

θ = θ1 + θ2 = 12 ω2 + 21 ω1 = 12 (ω1 + ω2 ) = 21 ω.

Thus, θ = 21 ω.
Case 3. The sides of θ lies on the same side of the diameter (see figure).
Consider the angle formed by θ0 + θ and ω 0 + ω. From Case 1, we know that θ0 = 21 ω and θ0 + θ =
1
2
(ω 0 + ω). By substitution,

θ0 + θ = 12 ω 0 + 12 ω
θ0 + θ =θ0 + 21 ω
θ = 12 ω.

17
In any case, θ = 12 ω.

21. Use induction to prove that n! > n2 for n ≥ 4.

Proof. Let n ≥ 4 be a positive integer and Pn be the statement that n! > n2 . Clearly, P4 is true
since 4! = 24 > 16 = 42 . We next show that if k is a positive integer and Pk is true, then Pk+1 is also
true. If Pk is true, then k! > k 2 . Observe that,

k! >k 2
(k + 1)k! >(k + 1)k 2
(k + 1)! >(k + 1)k 2 > (k + 1)(k + 1) = (k + 1)2 .

Thus, (k + 1)! > (k + 1)2 . This shows that Pk+1 is true. By the principle of Mathematical Induction,
Pn is true for all positive integers n ≥ 4.

22. The sum of the cubes of three consecutive integers is divisible by 9.

Proof. Let w, w + 1, and w + 2 be 3 consecutive integers. We want to show that

M = w3 + (w + 1)3 + (w + 2)3

is divisible by 9. Simplifying M gives

M =w3 + (w + 1)3 + (w + 2)3


=w3 + (w3 + 3w2 + 3w + 1) + (w3 + 6w2 + 12w + 8)
=3w3 + 9w2 + 15w + 9
=3w3 + 6w + 9w2 + 9w + 9
=3w(w2 + 2) + 9(w2 + w + 1).

For M to be divisible by 9, it suffices to show that 3w(w2 + 2) is divisible by 9. We consider 3 cases


on the value of w.
Case 1. w = 3k, k ∈ Z.
Thus, 3w(w2 + 2) = 3(3k)(w2 + 2) = 9k(w2 + 2) is divisible by 9.
Case 2. w = 3k + 1, k ∈ Z.
Thus, 3w(w2 +2) = 3w[(3k +1)2 +2] = 3w[(9k 2 +6k +1)+2] = 3w[3(3k 2 +2k +1)] = 9w(3k 2 +2k +1)
is divisible by 9.
Case 3. w = 3k + 2, k ∈ Z.
Thus, 3w(w2 +2) = 3w[(3k+2)2 +2] = 3w[(9k 2 +12k+4)+2] = 3w[3(3k 2 +4k+2)] = 9w(3k 2 +4k+2)
is divisible by 9.
In each of the following cases, we see that 3w(w2 + 2) is divisible by 9 and so M is divisible by 9.

.............................................................................................................................................................................................................................................................................................................................................................................................................................................................................................................................................................................................................................................................................................................................................................................................................

References: Prepared By: Jose B. Rosario and Arnold A. Eniego


K. H. Rosen, Discrete Mathematics and its Applications, McGraw-Hill Companies, Inc., 2012
E.A. Cabral, E.P. Delara-Tuprio, M.L.A.N. De Las Peñas, F.F. Francisco, I.J.L. Garces, R.M. Marcelo, J.F. Sarmiento,
Precalculus, Ateneo de Manila University Press, 2010
J. A. Gallian, Contemporary Abstract Algebra, Seventh Edition, Brooks/Cole, Cengage Learning, 2010
R. Hammack, Book of Proof, 2003
D. Burton, Elementary Number Theory, 5th Edition, McGraw-Hill, 2002

18

S-ar putea să vă placă și